Re: [obm-l] Problema de geometria plana

2006-04-26 Por tôpico Marcio M Rocha

[EMAIL PROTECTED] escreveu:


Srs,

O problema abaixo é o de número 55 do livro matematica para o
vestibular da UFMG
(geometria plana) do Prof Christiano Sena.

(sem acentos)
Num triangulo ABC, AB =8 cm e AC = 10cm. Pelo incentro do triangulo,
traca-se uma reta paralela
a BC, que intercepta AB em M e AC em N. O perimetro do triangulo AMN eh:
a) 16 b) 17 c) 18 d) 19 e) 20.

alguem sabe sua solução? o gabarito diz que é 20.

at

sarmento




Aqui na Oi Internet você ganha ou ganha. Além de acesso grátis com qualidade, 
ganha contas ilimitadas de email com 1 giga cada uma. Ganha 60 mega para hospedar 
sua página pessoal. Ganha flog, suporte grátis e muito mais. Baixe grátis o 
Discador em http://www.oi.com.br/discador e comece a ganhar.


Agora, se o seu negócio é voar na internet sem pagar uma fortuna, assine 
Oi Internet banda larga por apenas R$ 9,90. Clique em 
http://www.oi.com.br/bandalarga e aproveite essa bocada!



=
Instruções para entrar na lista, sair da lista e usar a lista em
http://www.mat.puc-rio.br/~nicolau/olimp/obm-l.html
=


 



Boa noite, Sarmento.

Seja I o incentro do triângulo. Sabe-se que med(MBI)=med(IBC) e que 
med(NCI)=med(ICB). Por outro lado, sendo MN paralelo a BC, tem-se que 
med(MIB)=med(IBC)=med(MBI) e med(NIC)=med(ICB)=med(NCI). Daí: MB = MI e 
NC = NI.


O perímetro de AMN é:

AM + MN + NA = AM + MI + IN + NA = AM + MB + NC + NA = AB + AC = 18.

Se algo estiver errado, leve em conta o horário.

Abraços,

Márcio.

=
Instruções para entrar na lista, sair da lista e usar a lista em
http://www.mat.puc-rio.br/~nicolau/olimp/obm-l.html
=


Re: [obm-l] Polinomio

2006-04-04 Por tôpico Marcio M Rocha

Klaus Ferraz escreveu:

Mostre que se a,b,c sao inteiros impares, a equacao ax^2+bx+c nao tem 
raiz racional.



Novidade no Yahoo! Mail: receba alertas de novas mensagens no seu 
celular. Registre seu aparelho agora! 
http://us.rd.yahoo.com/mail/br/tagline/mobile_alerts/*http://br.mobile.yahoo.com/mailalertas/ 





No virus found in this incoming message.
Checked by AVG Free Edition.
Version: 7.1.385 / Virus Database: 268.3.5/301 - Release Date: 4/4/2006
 

Leia o artigo de Eduardo Wagner, Paridade, que pode ser encontrado em 
http://www.obm.org.br/frameset-eureka.htm.


Márcio.
=
Instruções para entrar na lista, sair da lista e usar a lista em
http://www.mat.puc-rio.br/~nicolau/olimp/obm-l.html
=


Re: [obm-l] Re:[obm-l] Fatoração

2006-02-22 Por tôpico Marcio M Rocha

Salhab [ k4ss ] escreveu:



 
(a+b+c)^4 = 1

*fatorando*.. temos:
a^4 + b^4 + c^4 + 4 [(ab)^2 + (bc)^2 + (ac)^2 + 2abc] = 1
a^4 + b^4 + c^4 + 4 * 1/4 = 1
a^4 + b^4 + c^4 = 0
 

 
 

Sem querer ser chato, gostaria de fazer uma pequeníssima correção nas 
palavras: no caso, o correto é *expandindo*, e não fatorando.


Márcio.


=
Instruções para entrar na lista, sair da lista e usar a lista em
http://www.mat.puc-rio.br/~nicolau/olimp/obm-l.html
=


Re: [obm-l] altura_triangulo

2006-02-15 Por tôpico Marcio M Rocha

elton francisco ferreira escreveu:


ola pessoal da lista!
como resolver a questao que se segue?

Determine a altura nao relativa a base de um triangulo
isosceles de lados 10m, 10m e 12 m.








___ 
Yahoo! doce lar. Faça do Yahoo! sua homepage. 
http://br.yahoo.com/homepageset.html 


=
Instruções para entrar na lista, sair da lista e usar a lista em
http://www.mat.puc-rio.br/~nicolau/olimp/obm-l.html
=


 


1) Encontre a altura relativa à base;
2) Calcule a área do triângulo;
3) Sendo *h* a altura procurada e sendo a base (agora) igual a 10m, use 
a informação de 2) para calcular *h*.


Márcio.
=
Instruções para entrar na lista, sair da lista e usar a lista em
http://www.mat.puc-rio.br/~nicolau/olimp/obm-l.html
=


[obm-l] Ensino de matematica

2005-10-17 Por tôpico Marcio M Rocha
Gostaria de fazer uma pergunta ao pessoal da lista que lida com ensino, 
principalmente (mas não exclusivamente) a nivel fundamental e médio: por 
que o Brasil tem um desempenho tão bom nas competições internacionais de 
matemática mas, ao mesmo tempo, é tão mal avaliado no, vamos dizer, 
ensino normal? Pergunto isso porque há países que têm qualidade tanto na 
matemática olímpica quanto na matemática escolar.

Qual a sua opinião, professor Nicolau?
Desculpem se o assunto foge ao tema principal.

[]s,

Márcio.
=
Instruções para entrar na lista, sair da lista e usar a lista em
http://www.mat.puc-rio.br/~nicolau/olimp/obm-l.html
=


Re: [obm-l] Aritmética Progressiva

2005-10-08 Por tôpico Marcio M Rocha

Ilídio Leite escreveu:


olá...
 
alguém conhece o livro Aritmética Progressiva, de Antônio Trajano?

achei um exemplar num sebo mas estava encapado, não pude folheá-lo.
 
 
abraços a todos,

Ilídio Leite


Caro Ilídio,

Eu tinha um exemplar deste livro. Fiquei muito empolgado quando comprei 
porque o Elon (que ele me desculpe a intimidade) fala sempre muito bem 
deste livro. Bem, entrei em contato com o próprio Elon falando sobre o 
livro e coisa e tal e aí veio o balde de água fria: minha edição não era 
a que ele usara, ou seja, o exemplar que estava comigo já havia sofrido 
alterações e reduções. Havia comprado gato por lebre.


Se você conseguir uma permissão para dar uma olhada no interior do 
livro, procure se consta a definição de proporcionalidade. Se constar, 
compre o livro (e depois vou tentar uma cópia contigo!), caso contrário, 
não creio que o investimento valha a pena, mas aí é contigo.


Abraços,

Márcio.
=
Instruções para entrar na lista, sair da lista e usar a lista em
http://www.mat.puc-rio.br/~nicolau/olimp/obm-l.html
=


Re: [obm-l] Ajuda - Proporção

2005-10-01 Por tôpico Marcio M Rocha

admath escreveu:


Olá
 
Já li diversas teorias sobre proporcionalidade só que não consigo 
entender estes dois problemas de maneira alguma. Alguém pode me 
explicar de uma maneira bem didática?
 
1) Dividindo 70 em partes proporcionais a 2, 3 e 5, a soma entre a 
menor e a maior parte é quanto?
 
2) A proporção entre as medalhas de ouro, prata e bronze de um 
atleta é 3 : 4 : 7, respectivamente. Quantas medalhas de ouro, prata e 
bronze espera-se que esse atleta obtenha em 70 jogos, se essa 
proporção se mantiver e ele conquistar medalhas em todos os jogos?
 
-Posso falar que o método da regra de 3 é o mesmo processo quando 
lidamos com grandezas proporcionais?
 
obrigado.


1) Complementando a solução do Jefferson: Quando o problema menciona 
partes proporcionais', está implícito que são partes DIRETAMENTE 
proporcionais. Quando ele diz que vai dividir 70 em partes proporcionais 
a 2, 3 e 5, isso significa que há um valor constante (chamado constante 
de proporcionalidade) e que a primeira parte vale o dobro, a segunda o 
triplo e a terceira o quíntuplo dessa constante.


Esse problema também pode ser resolvido usando um método conhecido como 
falsa posição. A idéia é a seguinte:


Se as partes valessem 2, 3 e 5, a soma valeria 10. Como a soma das 
partes é 70, que é 7 vezes 10, cada parte deve ser, também, multiplicada 
por 7. Logo, cada parte vale 14, 21 e 35.


2) Usando a idéia da falsa posição:

Para ganhar 3 medalhas de ouro, 4 de prata e 7 de bronze ele deve 
disputar 3 + 4 + 7 = 14 jogos.

Se ele disputar 70 jogos (= 5 x 14) ele ganhará:

   5 x 3 = 15 medalhas de ouro.

   5 x 4 = 20 medalhas de prata.

   5 x 7 = 35 medalhas de bronze.

No primeiro problema a constante de proporcionalidade é 10, e no segundo, 5.

Espero que, com a solução do Jefferson e esses breves comentários, tudo 
tenha ficado mais claro para você.


[]s,

Márcio.
=
Instruções para entrar na lista, sair da lista e usar a lista em
http://www.mat.puc-rio.br/~nicolau/olimp/obm-l.html
=


Re: [obm-l] Mais uma questão da prova.

2005-09-02 Por tôpico Marcio M Rocha

Rejane escreveu:


Quem puder me ajudar, eu agradeço.

 


Abraços.

 


Rejane

 


Questão 08)

 

No triângulo *ABC* ao lado, se *M* e *N* são pontos médios e a área do 
triangulo *DMC* é 1 dm², então a área, em dm², no triangulo *ABD* é:


 

A) 3  B) 2   C) 2,5  D) 
1,5  E) 1,9



  *M*

 



  *D*

 



  *N*

 



  *B*

 



  *C*

 



  *A*

 

 














Rejane, por falta de tempo devo ter escrito excessivamente, mas aí vai.

   Se a área de *DMC* é igual a 1, a área de DMB também é, pois os dois 
triângulos considerados têm mesma base e mesma altura. Daí, *Área *de 
*BDC* = 2. Como D é o baricentro de *ABC*, *BD*/*DN* = 2, e, por 
conseqüência, *Área* de *BDC* / *Área* de *DCN* = 2, ou seja, *Área* de 
*DCN* = 1. Isso significa que *Área* de *BCN* = 2 + 1 = 3. A Área de 
*ABN* = 3, pois N é médio de *AC*. A área de *ABD* = 2/3 da área de 
*ABN*, ou seja:


   *Área* de *ABD* = 2.

   Dê uma conferida, por favor.

[]s,

Márcio.
=
Instruções para entrar na lista, sair da lista e usar a lista em
http://www.mat.puc-rio.br/~nicolau/olimp/obm-l.html
=


Re: [obm-l] Economia Matemática

2005-08-06 Por tôpico Marcio M Rocha

Maurício escreveu:


 Oi, sou eu de novo.

 Estou interessado em fazer uma pós na área de
Economia Matemática. Vocês sabem onde se faz pesquisa
de qualidade nessa área aqui no Brasil ou no Exterior?

 Abraços e obrigado,
 Maurício


__
Do You Yahoo!?
Tired of spam?  Yahoo! Mail has the best spam protection around 
http://mail.yahoo.com 
=

Instruções para entrar na lista, sair da lista e usar a lista em
http://www.mat.puc-rio.br/~nicolau/olimp/obm-l.html
=

 

Se você procura qualidade, penso que não se pode deixar de mencionar o 
IMPA, que tem cursos de mestrado e doutorado em Economia Matemática. 
Veja em www.impa.br.


Márcio.
=
Instruções para entrar na lista, sair da lista e usar a lista em
http://www.mat.puc-rio.br/~nicolau/olimp/obm-l.html
=


Re: [obm-l] geometria

2005-07-25 Por tôpico Marcio M Rocha

Desculpe-me, Saulo, mas os ângulos são congruentes sim. Veja:

QBP é um ângulo formado pela corda BP e pelo lado AB, que é tangente à 
circunferência. Logo, mede metade do arco menor BP. O ângulo PCB é um 
ângulo inscrito e também mede metade do arco menor BP. Uma argumentação 
parecida vale para os ângulos PCR e PBD.


Outra coisa: quem disse que BC é um diâmetro?

Dê uma conferida com cuidado, por favor.

[]s,

Márcio.



saulo nilson escreveu:


os angulos nao sao congruentes BC nao passa pelo centro da
circunferencia, BC e uma corda e nao um diametro.
Eu fiz achando o raio da circunferencia que e 13, dai vc acha os lados
e pela area do triangulo isosceles vc acha acha o valor da altura
pedida, mas na da uma resposta simples.

Eu projetei PQ e PR sobre os raios que unem o centro aos pontos de
tangencia da circunferencia ai obtive um quadrilatero que tem lados
r-9 e r-4 que e semelhante a AQPR, dai vc acha o raio que da 13, isso
esta certo ou errado?
On 7/20/05, Marcio M Rocha [EMAIL PROTECTED] wrote:
 


Eder Albuquerque escreveu:

   


Olá,

Gostaria de ajuda no seguinte problema: seja ABC um triângulo
isósceles, onde AB=AC são tangentes a uma circunferência e BC é uma
corda. Seja P um ponto sobre a circunferência anterior, interno ao
triângulo  ABC, tal que a distância de P a AB é 9 e a distância de P a
AC é 4. Encontre a distância de P a BC.

Não tô conseguindo resolver...

Grato,

Eder

__
Converse com seus amigos em tempo real com o Yahoo! Messenger
http://br.download.yahoo.com/messenger/

 


A resposta é 6.

Sejam Q, R e D os pés das perpendiculares, respectivamente a AB, AC e BC
por P.  Construa o triângulo BPC. Veja que os ângulos QBP e PCB são
conguentes. Daí, os triângulos retângulos QPB e  PDC  são semelhantes, e
podemos escrever que (PQ / PD) = (PB / PC).

Analogamente, os ângulos PCR e PBC são congruentes, donde vem a
semelhança dos triângulos retângulos PCR e PBD, e podemos escrever que
(PR / PD) = (PC / PB).

Logo, (PQ / PD) = (PD / PR), ou seja,  (PD)^2 = (PQ).(PR).

Pelo problema, PQ = 9  e  PR = 4. Assim, PD = 6.

Este problema consta do livro Challenging Problems in Geometry.

Um abraço,

Márcio.
=
Instruções para entrar na lista, sair da lista e usar a lista em
http://www.mat.puc-rio.br/~nicolau/olimp/obm-l.html
=

   



=
Instruções para entrar na lista, sair da lista e usar a lista em
http://www.mat.puc-rio.br/~nicolau/olimp/obm-l.html
=

 



=
Instruções para entrar na lista, sair da lista e usar a lista em
http://www.mat.puc-rio.br/~nicolau/olimp/obm-l.html
=


Re: [obm-l] geometria

2005-07-19 Por tôpico Marcio M Rocha

Eder Albuquerque escreveu:


Olá,
 
Gostaria de ajuda no seguinte problema: seja ABC um triângulo 
isósceles, onde AB=AC são tangentes a uma circunferência e BC é uma 
corda. Seja P um ponto sobre a circunferência anterior, interno ao 
triângulo  ABC, tal que a distância de P a AB é 9 e a distância de P a 
AC é 4. Encontre a distância de P a BC.
 
Não tô conseguindo resolver...
 
Grato,
 
Eder


__
Converse com seus amigos em tempo real com o Yahoo! Messenger
http://br.download.yahoo.com/messenger/


A resposta é 6.

Sejam Q, R e D os pés das perpendiculares, respectivamente a AB, AC e BC 
por P.  Construa o triângulo BPC. Veja que os ângulos QBP e PCB são 
conguentes. Daí, os triângulos retângulos QPB e  PDC  são semelhantes, e 
podemos escrever que (PQ / PD) = (PB / PC).


Analogamente, os ângulos PCR e PBC são congruentes, donde vem a 
semelhança dos triângulos retângulos PCR e PBD, e podemos escrever que 
(PR / PD) = (PC / PB).


Logo, (PQ / PD) = (PD / PR), ou seja,  (PD)^2 = (PQ).(PR).

Pelo problema, PQ = 9  e  PR = 4. Assim, PD = 6.

Este problema consta do livro Challenging Problems in Geometry.

Um abraço,

Márcio.
=
Instruções para entrar na lista, sair da lista e usar a lista em
http://www.mat.puc-rio.br/~nicolau/olimp/obm-l.html
=


Re: [obm-l] geometria plana

2005-07-19 Por tôpico Marcio M Rocha

elton francisco ferreira escreveu:


Olá pessoal, ai vai algumas questoes que começei a a
resoluçao mas nao consigo terminar. Se vcs puderem me
ajudarem.

1 - Num triangulo retangulo ABC, sabe-se que a área
vale 2s e que a razão entre os catetos é b/c=k.
Calcule seus lados.

2 – A diferença entre os catetos de um triangulo
retângulo é d e a área é s. Determine os catetos para:
d = 17 e s = 84
3 – Num triangulo retângulo, a hipotenusa é a e a
altura relativa a ela é h. Calcule os catetos para : a
= 25 e h = 12






___ 
Yahoo! Acesso Grátis - Internet rápida e grátis. 
Instale o discador agora! http://br.acesso.yahoo.com/

=
Instruções para entrar na lista, sair da lista e usar a lista em
http://www.mat.puc-rio.br/~nicolau/olimp/obm-l.html
=

 

1- A área é a metade de bc, ou seja, bc = 4s. Multiplicando ambos os 
membros de b/c = k por bc, vem


b^2 = bck, ou seja, b = sqrt(4ks), ou b = 2.sqrt(ks). Daí, c = 
2.sqrt(ks) / k


2- b = 24 e c = 7. Como o produto dos catetos é o dobro da área, temos 
que bc = 168. Como b - c = 17, você cai numa eq do 2o grau.


3- b = 20 e c = 15. Veja que b^2 + c^2 = 625 e bc = 300. Daí, b + c = 
35, e você monta uma equação do 2o grau.


Abraços,

Márcio.
=
Instruções para entrar na lista, sair da lista e usar a lista em
http://www.mat.puc-rio.br/~nicolau/olimp/obm-l.html
=


Re: [obm-l] Problema

2005-06-28 Por tôpico Marcio M Rocha

Lincoln escreveu:


Alguém pode me dar uma ajuda neste problema?
 

Seja /ABCD/ um retângulo de lados /AB/ = 4 e /BC/ =3.  A perpendicular 
à diagonal /BD/ traçada por /A/ corta /BD/ no ponto /H/. Chamamos de 
/M/ o ponto médio de /BH/ e de /N/ o ponto médio de /CD/. Calcule a 
medida do segmento /MN/.


 

 


Lincoln,
Escrevi rápido e sem muita organização. Veja se você entende e se está 
tudo OK.


BD = 5 e AH = 2,4. A perpendicular ao segmento AH, passando por M, 
intersecta AB no ponto P. Desse modo, MP = 1,2 e P é o médio de AB. Os 
segmentos PN e BD cortam-se ao meio, num ponto Q.  Seja R um ponto de 
BD, simétrico de M em relação a Q. Os triângulos PMQ e NRQ são 
congruentes. Desse modo, MN^2 = RN^2 + RM^2, ou seja, MN^2 = (1,2)^2 + 
(1,8)^2. Daí, MN = 3*sqrt(13)/5.


[]s,

Márcio.
=
Instruções para entrar na lista, sair da lista e usar a lista em
http://www.mat.puc-rio.br/~nicolau/olimp/obm-l.html
=


Re: [obm-l] Dúvida elementar

2005-06-09 Por tôpico Marcio M Rocha

Luiz Ernesto Leitao escreveu:

Lendo o livro de Análise do Djairo Guedes ele pediu que se provasse a 
seguinte afirmativa elementar:

Prove que  p ( p natural) é par, se e somente se, p^2 for par.

__
Converse com seus amigos em tempo real com o Yahoo! Messenger
http://br.download.yahoo.com/messenger/


1a parte: p é par = p^2 é par

p é par = p = 2n

Donde se conclui que p^2 = 4(n^2) = 2(2n^2) que é par

2a parte: p^2 é par = p é par

Prove a contrapositiva: p é ímpar = p^2 é ímpar

p é ímpar = p = 2n +1

Daí,

p^2 = (2n + 1)^2 = 4n^2 + 4n + 1 = 2(2n^2 + 2n) + 1 que é ímpar.

Márcio.
=
Instruções para entrar na lista, sair da lista e usar a lista em
http://www.mat.puc-rio.br/~nicolau/olimp/obm-l.html
=


Re: [obm-l] Problemas russos

2005-05-24 Por tôpico Marcio M Rocha

fabiodjalma escreveu:

Onde você os encontrou? 



Em (09:21:52), obm-l@mat.puc-rio.br escreveu: 



 

Bom dia a todos! 

Encontrei 100 problemas russos traduzidos pelo Paulo Santa Rita e estou 
tentando resolvê-los. Gostaria de uma idéia para o seguinte: 

É dado um retangulo ABCD com o comprimento da diagonal AC valendo L. 
Quatro círculos com centros em A, B, C e D e raios respectivamente 
iguais a a, b, c e d, sao tais que : L  a + c , a + c = b + d. 
Prove que se pode inscrever um circulo no quadrilatero formado pelas 
interseccões entre duas tangentes comuns externas ao circulos A e C e 
duas tangentes comuns externas aos circulos B e D. 

Um grande abraço. 

[]s, 

Márcio. 
= 
Instruções para entrar na lista, sair da lista e usar a lista em 
http://www.mat.puc-rio.br/~nicolau/olimp/obm-l.html 
= 

-- 



 


Oi, Fábio,

Os problemas estão em http://www.mat.puc-rio.br/~nicolau/psr/ 
http://www.mat.puc-rio.br/%7Enicolau/psr/


Um abraço.

Márcio.
=
Instruções para entrar na lista, sair da lista e usar a lista em
http://www.mat.puc-rio.br/~nicolau/olimp/obm-l.html
=


Re: [obm-l] Re: [obm-l] Função

2005-05-03 Por tôpico Marcio M Rocha
Viviane Silva escreveu:

Como se resolve uma função do tipo. Este não é o exercício mas é 
parecido com este
 
1) f(3x+1)=x^2+3x+25
g(x+1)=2x+1
Encontre f(g(-1))
 
Grata


MSN Busca: fácil, rápido, direto ao ponto. Encontre o que você quiser. 
Clique aqui. http://g.msn.com/8HMABR/2740??PS=47575 
= 
Instruções para entrar na lista, sair da lista e usar a lista em 
http://www.mat.puc-rio.br/~nicolau/olimp/obm-l.html 
= 
g(-1) = g(-2 + 1) = 2*(-2) + 1 = -4 + 1 = -3
f(-3) = f(3*(-4/3) + 1) = (-4/3)^2 + 3*(-4/3) + 25 = 16/9 - 4 + 25 = (16 
+ 189)/9 = 205/9

Márcio.
=
Instruções para entrar na lista, sair da lista e usar a lista em
http://www.mat.puc-rio.br/~nicolau/olimp/obm-l.html
=


[obm-l] Princípio das gavetas

2005-03-30 Por tôpico Marcio M Rocha
Olá, pessoal!
Antes de mais nada, obrigado ao Cláudio e ao Qwert pela solução do problema.
Como estou com um tempinho livre, vou escrever uns pensamentos muito 
rápido. Vejam se tem algum fundamento.

Em 39 números consecutivos, formo 13 conjuntos disjuntos, cada qual com 
3 números consecutivos. Obviamente, um deles é múltiplo de 3, o que 
implica que a soma dos algarismos de um elemento de cada um dos 13 
conjuntos é igual a 3k.

Tomando esse elemento de cada um dos 13 conjuntos, tenho 13 múltiplos 
consecutivos de 3, ou seja, 13 números cuja soma dos algarismos é um 
múltiplo de 3. Como são 13 números e todos são consecutivos, tô pensando 
se existe um meio de garantir que uma dessas somas também é múltiplo de 11.

O que vocês acham?
[]s,
Márcio.

=
Instruções para entrar na lista, sair da lista e usar a lista em
http://www.mat.puc-rio.br/~nicolau/olimp/obm-l.html
=


Re: [obm-l] Princípio das gavetas

2005-03-30 Por tôpico Marcio M Rocha
Marcio M Rocha escreveu:
Em 39 números consecutivos, formo 13 conjuntos disjuntos, cada qual 
com 3 números consecutivos. Obviamente, um deles é múltiplo de 3, o 
que implica que a soma dos algarismos de um elemento de cada um dos 13 
conjuntos é igual a 3k.

Tomando esse elemento de cada um dos 13 conjuntos, tenho 13 múltiplos 
consecutivos de 3, ou seja, 13 números cuja soma dos algarismos é um 
múltiplo de 3. Como são 13 números e todos são consecutivos, tô 
pensando se existe um meio de garantir que uma dessas somas também é 
múltiplo de 11.

O que vocês acham?
[]s,
Márcio.

=
Instruções para entrar na lista, sair da lista e usar a lista em
http://www.mat.puc-rio.br/~nicolau/olimp/obm-l.html
=
Eu respondo a mim mesmo: o fato dos números serem múltiplos consecutivos 
de 3 não significa que as somas dos algarismos sejam números consecutivos.

Desculpem-me.
[]s,
Márcio.
=
Instruções para entrar na lista, sair da lista e usar a lista em
http://www.mat.puc-rio.br/~nicolau/olimp/obm-l.html
=


Re: [obm-l] Problema 1 do primeiro nível da X Olimpíada de Maio

2005-03-30 Por tôpico Marcio M Rocha
Daniel S. Braz escreveu:
Pessoal,
Uma dúvida no problema 1 do primeiro nível da X Olimpíada de Maio
(Eureka! número 20)
Xavier multiplica quatro dígitos, não necessariamente distintos, e obtém um
número terminado em 7. Determine quanto pode valer a soma dos quatros dígitos
multiplicados por Xavier. Dê todas as possibilidades.
então devemos ter algo do tipo: a.b.c.d = xxx7
A dúvida é: Qualquer número terminado em 9 multiplicado por 3 dará um
número terminado em 7
1.1.1.7 = 7 - 10
1.1.3.9 = 27 - 14
1.1.3.19 = 57 - 24
1.1.3.29 = 87 - 34
1.1.3.39 = 117 - 44
Então...como calcular todas as possibilidades...não entendi...
 

Oi, Daniel
Veja que o enunciado diz que ele multiplica quatro dígitos, ou seja, 
algarismos. Assim, as 3 últimas possibilidades que você mostrou estão 
descartadas.

[]s,
Márcio.
=
Instruções para entrar na lista, sair da lista e usar a lista em
http://www.mat.puc-rio.br/~nicolau/olimp/obm-l.html
=


[obm-l] Principio das Gavetas

2005-03-29 Por tôpico Marcio M Rocha
Bom dia, pessoal!
Gostaria de conferir uma solução do seguinte problema: Mostre que 
existe um múltiplo de 1997 que possui todos os dígitos iguais a 1

Usando o princípio das gavetas é possível mostrar que todo número 
natural possui um múltiplo que se escreve usando apenas os dígitos 0 e 
1, de modo que haja uma seqüência de /p/ 1's seguida de /q/ 0's.

Seja N = 111...1000...0 um múltiplo de 1997. Como N = (111...1) * 
(10^/q/) e 1997 não divide 10^/q, /conclui-se que 1997 divide 111...1.

Tá tudo Ok?
Aproveitando a oportunidade, gostaria de uma sugestão no problema 
seguinte: Prove que em qualquer seqüência de 39 números naturais 
consecutivos existe ao menos um número cuja soma dos algarismos é 
divisível por 11.

[]s,
Márcio.
=
Instruções para entrar na lista, sair da lista e usar a lista em
http://www.mat.puc-rio.br/~nicolau/olimp/obm-l.html
=


Re: [obm-l] Principio das Gavetas

2005-03-29 Por tôpico Marcio M Rocha
claudio.buffara escreveu:
 
*De:* 	[EMAIL PROTECTED]

*Para:* obm-l@mat.puc-rio.br
*Cópia:*
*Data:* Tue, 29 Mar 2005 08:44:28 -0300
*Assunto:*  [obm-l] Principio das Gavetas
 Bom dia, pessoal!

 Gostaria de conferir uma solução do seguinte problema: Mostre que
 existe um múltiplo de 1997 que possui todos os dígitos iguais a 1

 Usando o princípio das gavetas é possível mostrar que todo número
 natural possui um múltiplo que se escreve usando apenas os dígitos 0 e
 1, de modo que haja uma seqüência de /p/ 1's seguida de /q/ 0's.

 Seja N = 111...1000...0 um múltiplo de 1997. Como N = (111...1) *
 (10^/q/) e 1997 não divide 10^/q, /conclui-se que 1997 divide 111...1.

 Tá tudo Ok?

Pra mim, está.
 
Uma demonstração alternativa usa o teorema de Euler e leva em conta 
que mdc(1997,10) = mdc(1997,9) = 1.
Nesse caso, pondo k = Phi(1997), teremos 10^k == 1 (mod 1997) ==
1997 | 10^k - 1 = 999999  (k algarismos 9) = 9*111...111.
Como 1997 é primo com 9, concluímos que 1997 | 111...111.
 
 
 Aproveitando a oportunidade, gostaria de uma sugestão no problema
 seguinte: Prove que em qualquer seqüência de 39 números naturais
 consecutivos existe ao menos um número cuja soma dos algarismos é
 divisível por 11.

Esse parece interessante. Acho que vale a pena fazer umas simulações 
no Excel pra ver se você acha alguma periodicidade ou lei de formação. 
Se eu achar alguma coisa te falo.
 
[]s,
Claudio.
 
Cláudio,
Obrigado pela solução alternativa e pela dica.
[]s,
Márcio.
=
Instruções para entrar na lista, sair da lista e usar a lista em
http://www.mat.puc-rio.br/~nicolau/olimp/obm-l.html
=


[obm-l] Problemas russos

2005-03-28 Por tôpico Marcio M Rocha
Bom dia a todos!
Encontrei 100 problemas russos traduzidos pelo Paulo Santa Rita e estou 
tentando resolvê-los. Gostaria de uma idéia para o seguinte:

É dado um retangulo ABCD com o comprimento da diagonal AC valendo L. 
Quatro círculos com centros em A, B, C e D  e raios respectivamente 
iguais a a, b, c e d, sao tais que : L  a + c ,  a + c = b + d. 
Prove que se pode inscrever um circulo no quadrilatero formado pelas 
interseccões entre duas tangentes comuns externas ao circulos A e C e 
duas tangentes comuns externas aos circulos B e D.

Um grande abraço.
[]s,
Márcio.
=
Instruções para entrar na lista, sair da lista e usar a lista em
http://www.mat.puc-rio.br/~nicolau/olimp/obm-l.html
=


Re: [obm-l] ITA

2005-03-26 Por tôpico Marcio M Rocha
fgb1 escreveu:
Um aluno me pediu p/ fazer essa questão e disse que era do ITA. Não 
encntrei solução. Queria saber se alguem conhece e pode confirmar se o 
enunciado está correto.
 
3^2x + 5^2x - 15^x = 0
 
A idéia é dividir tudo por 15^x e, por meio de artifício, cair numa eq 
do 2o grau. Só que, fazendo tudo isso, você vai cair numa eq sem solução 
real. Deve haver algo de errado no enunciado.

[]s,
Márcio.
=
Instruções para entrar na lista, sair da lista e usar a lista em
http://www.mat.puc-rio.br/~nicolau/olimp/obm-l.html
=


[Fwd: Re: En:[obm-l] Fatorial]

2005-03-24 Por tôpico Marcio M Rocha
Desculpem a todos, mais uma vez, as duas besteiras que escrevi: a 
desigualdade absurda e minimilidade. No afã de resolver o problema, 
fiquei cego a algo tão claro.

Bem, então não dá para escrever uma seqüência de desigualdades partindo 
de b elevado a (b - 2) e chegando a (b - 1) elevado a (b - 1)?

[]s,
Márcio.
---BeginMessage---
claudio.buffara escreveu:

 Além disso, Cláudio, também posso fazer:

 b elevado a (b - 2) = (b - 1) elevado a (b - 2)
 
Epa! Tem certeza?
 
 
CARAMBA! QUE BESTEIRA! Deleta esse negócio daí!
---End Message---


Re: [obm-l] Questõ de raiz quadrada

2005-03-23 Por tôpico Marcio M Rocha
Robÿe9rio Alves escreveu:
Escreva as implicações lógicas que correspondem à resolução da equação 
rsqt x + 6 = x, veja quais são reversíveis e explique o aparecimento 
de raízes estranhas. Faça o mesmo com a equação rsqt x + 7 =  x.

__
Converse com seus amigos em tempo real com o Yahoo! Messenger
http://br.download.yahoo.com/messenger/
Vamos lá,
sqrt(x) + 6 = x   = (1)
= sqrt(x) = x - 6   = (2)
 = (sqrt(x))² = (x - 6)² = (3)
= x = x² - 12x + 36 = (4)
= x² - 13x + 36 = 0 = (5)
= x = 9 ou x = 4= (6)
Observe que x = 4 não é raiz da equação original. Veja que de (2) para 
(3) não há uma equivalência, mas apenas uma implicação, ou seja, de (2) 
concluo (3) mas de (3) concluo (2) ou concluo sqrt(x) = 6 - x. Daí vem o 
aparecimento da raiz estranha igual a 4.

Agora, você faz com a outra.
[]s,
Márcio.
=
Instruções para entrar na lista, sair da lista e usar a lista em
http://www.mat.puc-rio.br/~nicolau/olimp/obm-l.html
=


[obm-l] Lugar Geométrico

2005-03-23 Por tôpico Marcio M Rocha
Boa noite, gente.
Estava observando uma figura que me sugeriu uma parábola e pensei na 
seguinte questão: Dada uma parábola e sua diretriz, determine o lugar 
geométrico dos pontos médios dos segmentos perpendiculares à diretriz e 
que têm, como extremidades, a diretriz e a parábola.

[]s,
Márcio.
=
Instruções para entrar na lista, sair da lista e usar a lista em
http://www.mat.puc-rio.br/~nicolau/olimp/obm-l.html
=


Re: [obm-l] Re:[obm-l] Lugar Geométrico

2005-03-23 Por tôpico Marcio M Rocha
claudio.buffara escreveu:
Seja a parabola y = 2ax^2 e a reta y = 2bx + 2c.
 
O ponto medio de um segmento vertical ligando os pontos (x,2bx+2c) da 
reta e (x,2ax^2) da parabola eh (x,ax^2+bx+c).
 
Logo, o LG desses pontos medios eh uma parabola.
 
Repare que a reta nao precisa ser a diretriz e, de fato, nao precisa 
nem ser paralela a diretriz. Soh nao pode ser perpendicular a esta.
 
[]s,
Claudio.
 
*De:* 	[EMAIL PROTECTED]

*Para:* obm-l@mat.puc-rio.br
*Cópia:*
*Data:* Wed, 23 Mar 2005 20:47:35 -0300
*Assunto:*  [obm-l] Lugar Geométrico
 Boa noite, gente.

 Estava observando uma figura que me sugeriu uma parábola e pensei na
 seguinte questão: Dada uma parábola e sua diretriz, determine o lugar
 geométrico dos pontos médios dos segmentos perpendiculares à diretriz e
 que têm, como extremidades, a diretriz e a parábola.

 []s,

 Márcio.
 
=
 Instruções para entrar na lista, sair da lista e usar a lista em
 http://www.mat.puc-rio.br/~nicolau/olimp/obm-l.html
 
=

Jóia, tá claro. É um probleminha muito sem graça...
[]s,
Márcio.
=
Instruções para entrar na lista, sair da lista e usar a lista em
http://www.mat.puc-rio.br/~nicolau/olimp/obm-l.html
=


[obm-l] Círculo e elipse

2005-03-23 Por tôpico Marcio M Rocha
Oi, pessoal.
Vamos ver se esse é um pouco melhor...
Estou segurando um CD diante dos meus olhos, de modo que, ao olhá-lo, 
vejo um círculo. Agora passo a rotacioná-lo em torno de seu diâmetro. É 
possível provar que a figura que passo a ver é uma elipse?

[]s,
Márcio.
=
Instruções para entrar na lista, sair da lista e usar a lista em
http://www.mat.puc-rio.br/~nicolau/olimp/obm-l.html
=


Re: [obm-l] Questão de peso ( Não entendi )

2005-03-22 Por tôpico Marcio M Rocha
Robÿe9rio Alves escreveu:
DAdos n ( n maior ou igual do que 2 ) objetos de pesos distintos, 
prove que é possivel determinar qual o mais pesado fazendo 2n - 3 
pesagens em uma balança de pratos. É esse número mínimo de pesagens 
que permitem determinar o mais leve e o mais pesado ?

__
Converse com seus amigos em tempo real com o Yahoo! Messenger
http://br.download.yahoo.com/messenger/
O Cláudio já resolveu essa questão. Dá uma olhada no arquivo da lista.
=
Instruções para entrar na lista, sair da lista e usar a lista em
http://www.mat.puc-rio.br/~nicolau/olimp/obm-l.html
=


Re: [obm-l] Poderiam resolver essa de números naturais =3F?=

2005-03-22 Por tôpico Marcio M Rocha
Robÿe9rio Alves escreveu:
Seja P(n) uma propriedade relativa ao número natural n. Suponha que P( 
1 ) , P ( 2 ) são verdadeiras e que, para qualquer n pertencente a IN, 
a verdade de P n ) e P ( n + 1 ).Prove que P ( n ) é verdadeira para 
todo n pertencente a IN


Yahoo! Mail 
%20http://us.rd.yahoo.com/mail/br/taglines/*http://mail.yahoo.com.br/ 
- Com *250MB* de espaço. Abra sua conta! 
Essa questão também já foi resolvida.
=
Instruções para entrar na lista, sair da lista e usar a lista em
http://www.mat.puc-rio.br/~nicolau/olimp/obm-l.html
=


[obm-l] Fatorial

2005-03-22 Por tôpico Marcio M Rocha
Pessoal, boa noite!
Aqui vai um probleminha: Prove que (n²)!  (n!)² para todo n  1.
[]s,
Márcio.
=
Instruções para entrar na lista, sair da lista e usar a lista em
http://www.mat.puc-rio.br/~nicolau/olimp/obm-l.html
=


[obm-l] Inducao(Parte 2)

2005-03-18 Por tôpico Marcio M Rocha
Penso que resolvi a segunda parte da questão:

Sabe-se que ((n + 1)/n) = n, logo:

a) Se n = 3
(4/3)³  3 (a igualdade obviamente não vale)
4³  3 elevado a 4
Elevando a 1/12 ...
4¹/4  3¹/3

b) Se n = 4
(5/4) elevado a 4  4
5 elevado a 4  4 elevado a 5
Elevando a 1/20 ...
5 elevado a 1/5  4 elevado a 1/4

Procedendo de modo análogo para n=5,6,..., constrói-se a cadeia abaixo:
3¹/3  4¹/4  5¹/5  6¹/6  ...

Tudo confere?

Márcio.

=
Instruções para entrar na lista, sair da lista e usar a lista em
http://www.mat.puc-rio.br/~nicolau/olimp/obm-l.html
=


[obm-l] Jogos e matematica

2005-03-18 Por tôpico Marcio M Rocha
Alguém sabe onde posso encontrar material sobre a matemática do Resta 
um e do Cubo Mágico? Se for material na internet, melhor ainda.

Obrigado a todos.
Márcio.
=
Instruções para entrar na lista, sair da lista e usar a lista em
http://www.mat.puc-rio.br/~nicolau/olimp/obm-l.html
=


[obm-l] Indução

2005-03-17 Por tôpico Marcio M Rocha
Boa noite, pessoal.
A questão abaixo também consta do Vol. 1 de A Matemática do Ensino Médio. 
Ela tem duas partes, das quais fiz a primeira. Gostaria de pedir que alguém 
verificasse se está tudo OK.

Parte 1) Prove que ((n + 1)/n) elevado a n =n para todo n=3.

Para n = 3 temos (4/3)³ =3

Solução
Supondo verdadeira para algum k3:

((k + 1)/k) elevado a k =k

Multiplico a desigualdade acima por ((k + 1)/k) e obtenho

((k + 1)/k)elevado a (k + 1) = k + 1

Só que quando k  3, (k + 2)/(k + 1) = (k + 1)/k, e daí:

((k + 2)/(k + 1)) elevado a (k + 1)  = ((k + 1)/k) elevado a (k + 1)

Logo (((k + 1) + 1)/(k + 1)) elevado a (k + 1) = k + 1

Parte 2) Use esse fato para mostrar que a seqüência

1, 2¹/2, 3¹/3, 4¹/4, ...

é decrescente a partir do 3o termo.

Esta parte ainda está saindo.

Desculpem se são questões triviais para vocês.

Abraços.

Márcio.

=
Instruções para entrar na lista, sair da lista e usar a lista em
http://www.mat.puc-rio.br/~nicolau/olimp/obm-l.html
=


[obm-l] Teorema de Cantor

2005-03-16 Por tôpico Marcio M Rocha
Pra ver se finalmente eu aprendo alguma coisa, estou resolvendo os problemas 
do volume 1 do livro A Matemática do Ensino Médio, do
Elon/PC/Wagner/Morgado. Ainda não consegui o seguinte (para quem tem o livro, 
é o exercício 20 do capítulo 1):

Prove o Teorema de Cantor: se A é um conjunto e P(A) é o conjunto das partes 
de A, não existe uma função f : A--P(A) que seja sobrejetiva.

Muito obrigado.

Márcio


=
Instruções para entrar na lista, sair da lista e usar a lista em
http://www.mat.puc-rio.br/~nicolau/olimp/obm-l.html
=


Re: (x+1)^p - x^p - 1 Era:[obm-l] Problemas diversos

2005-03-14 Por tôpico Marcio M Rocha
Dirichlet, você há de reconhecer, também, que calcular isso no braço em 
casa, com tempo livre, é uma coisa, mas numa prova de 20 questões, com 
tempo limitado, e tendo outras quetões de nível parecido, já é algo um 
pouco fora da realidade. Sendo assim, uma questão desse tipo constar 
numa prova de 8a série, sem as ferramentas adequadas para a sua solução, 
é meio despropositado.

Johann Peter Gustav Lejeune Dirichlet wrote:
Detalhes apenas...
1- Se alguem ai nao sabe binomio de Newton (algo
perdoavel para uma oitava serie, e preciso
reconhecer), basta calcular no braco!
2- Algo assim ja esteve na IMO...
 

   

 

   

 

=
 

Instruções para entrar na lista, sair da lista e
usar a lista em
http://www.mat.puc-rio.br/~nicolau/olimp/obm-l.html
   

=
 

__
Converse com seus amigos em tempo real com o Yahoo! Messenger 
http://br.download.yahoo.com/messenger/ 
=
Instruções para entrar na lista, sair da lista e usar a lista em
http://www.mat.puc-rio.br/~nicolau/olimp/obm-l.html
=

 

=
Instruções para entrar na lista, sair da lista e usar a lista em
http://www.mat.puc-rio.br/~nicolau/olimp/obm-l.html
=


Re: (x+1)^p - x^p - 1 Era:[obm-l] Problemas diversos

2005-03-14 Por tôpico Marcio M Rocha
Johann,
Acho que não faz sentido prolongar mais essa discussão. Infelizmente, 
coisas desse tipo acontecem...

Márcio.
Johann Peter Gustav Lejeune Dirichlet wrote:
Ahn?? Como isso foi parar numa prova de 20 questoes? E
com tempo?? Eu devo concordar contigo, nesse
(des)proposito.
Alias, quais sao as fontes deste troço?
--- Marcio M Rocha [EMAIL PROTECTED] wrote:
 

Dirichlet, você há de reconhecer, também, que
calcular isso no braço em 
casa, com tempo livre, é uma coisa, mas numa prova
de 20 questões, com 
tempo limitado, e tendo outras quetões de nível
parecido, já é algo um 
pouco fora da realidade. Sendo assim, uma questão
desse tipo constar 
numa prova de 8a série, sem as ferramentas adequadas
para a sua solução, 
é meio despropositado.

   





Yahoo! Mail - Com 250MB de espaço. Abra sua conta! http://mail.yahoo.com.br/
=
Instruções para entrar na lista, sair da lista e usar a lista em
http://www.mat.puc-rio.br/~nicolau/olimp/obm-l.html
=
 

=
Instruções para entrar na lista, sair da lista e usar a lista em
http://www.mat.puc-rio.br/~nicolau/olimp/obm-l.html
=


Re: [obm-l] Determine o algarismo

2005-03-14 Por tôpico Marcio M Rocha
O Nicolau já matou a questão, mas só como observação para os mais novos, 
isso era ensinado no primário, há alguns anos, com o nome de noves 
fora. Era uma das maneiras de se tirar a prova real de uma conta.

Nicolau C. Saldanha wrote:
On Mon, Mar 14, 2005 at 11:59:24AM -0800, Davidson Lima wrote:
 

Determine o algarismo A no produto
(9966334)*(9966332)=99327A93466888.
   

Seja a = 9966334, b = 9966332.
Temos a = 4 (mod 9), b = 2 (mod 9)
donde ab = 8 (mod 9)
Assim 9+9+3+2+7+A+9+3+4+6+6+8+8+8 = 1+A = 8 (mod 9)
donde A = 7.
O maple confere: ab = 99327793466888.
[]s, N.
=
Instruções para entrar na lista, sair da lista e usar a lista em
http://www.mat.puc-rio.br/~nicolau/olimp/obm-l.html
=
 

=
Instruções para entrar na lista, sair da lista e usar a lista em
http://www.mat.puc-rio.br/~nicolau/olimp/obm-l.html
=


Re: [obm-l] raciocinio logico

2005-03-13 Por tôpico Marcio M Rocha
Anna,
Essa questão consta do volume 1 do livro Fundamentos da Matemática 
Elementar, do Iezzi e outros. Na edição que usei na escola (na década 
de 80) ele constava dos testes no final do livro. E, se a memória não me 
falha, a resposta lá é 9. Ou seja, mesmo que você quisesse desconsiderar 
todas as soluções que foram apresentadas inequivocamente, há 
discrepância entre o gabarito do professor e o do livro.

Faça o que o Guilherme falou. Peça que o professor te convença que a 
resposta é 11 (o que ele não vai conseguir, segundo tudo que foi 
mostrado aqui), mas não se deixe levar por um decreto.

Abraços.
Márcio.
Anna Luisa wrote:
Oi gente!
Obrigadão pelo interesse, mas olha só o gabarito dá a resposta como 11 
dias ! Eu num acho de jeito nenhum e o prof diz que o gabarito tá 
certo! rsrsrsrs
[]s
Anninha.
=
Instruções para entrar na lista, sair da lista e usar a lista em
http://www.mat.puc-rio.br/~nicolau/olimp/obm-l.html
=


Re: [obm-l] QuestÃo de potencia

2005-03-13 Por tôpico Marcio M Rocha
Se você quer o resultado, usei o Maple e achei o pequeno número abaixo:
1577721810843758121888376491957973652876016166525299413111238457596825
Agora, para provar que termina com um número divisível por 5, é só fazer 
o seguinte:

* 1^99 termina em 1.
* 2^99 termina em 8, porque o último algarismo das potências de 2, a 
partir do expoente 1, repete-se de 4 em 4. Como 99 = 4 x 24 + 3, o 
último algarismo de 2^99 é o mesmo de 2^3.
* 3^99 termina em 7. O último algarismo das potências de 3, a partir do 
expoente 1, repete-se de 3 em 3. Então o último algarismo de 3^99 é o 
mesmo de 3^3.
* 4^99 = 2^198, que termina em 4.
* 5^99 termina em 5.
Logo, a expressão termina em 5.

[]s,
Márcio.
Robÿe9rio Alves wrote:
Qual o resultado da expressão 1^99 + 2^99 + 3^99 + 4^99 + 5^99 e prove 
que o resultado termina com um número divisível por 5.


Yahoo! Mail 
%20http://us.rd.yahoo.com/mail/br/taglines/*http://mail.yahoo.com.br/ 
- Com *250MB* de espaço. Abra sua conta! 
=
Instruções para entrar na lista, sair da lista e usar a lista em
http://www.mat.puc-rio.br/~nicolau/olimp/obm-l.html
=


Re: (x+1)^p - x^p - 1 Era:[obm-l] Problemas diversos

2005-03-13 Por tôpico Marcio M Rocha
Valeu pela ajuda nas duas questões, Cláudio.
Nessa questão específica eu não queria usar o binômio porque ela consta 
de uma prova de 8a série. Mas, pelo visto, não tem jeito...

Obrigado também pela dica do programa.
Márcio.
claudio.buffara wrote:
 
*De:* 	[EMAIL PROTECTED]

*Para:* obm-l@mat.puc-rio.br obm-l@mat.puc-rio.br
*Cópia:*
*Data:* Sat, 12 Mar 2005 12:16:17 -0300
*Assunto:*  [obm-l] Problemas diversos
 Boa tarde a todos!

 Gostaria de uma ajuda com os seguintes problemas (não é necessário
 resolver, só uma idéia já é o bastante)

 1) Se é que é possível, como fatorar (x + y)^7 - x^7 - y^7 sem usar
 expansão binomial?

Eu diria que eh possivel, mas usando o binomio e um pouquinho de 
braco, chega-se a fatoracao 7xy(x+y)(x^2+xy+y^2)^2.
 
Alias, isso me fez pensar nos polinomios F_p(x) = (x+1)^p - x^p - 1, 
com p primo.
 
Para p = 2, 3 e 5, as fatoracoes sao faceis. Respectivamente:
2x, 3x(x+1) e 5x(x+1)(x^2+x+1).
Para p = 7 eh soh usar o resultado acima: 7x(x+1)(x^2+x+1)^2.
 
Usando o Pari-GP - que alias estah com um upgrade facilimo de se 
intalar (veja o site: http://pari.math.u-bordeaux.fr/download.html e 
instale isso aqui:

  Binary distributions (development)
* Self-installing *Windows* binary: |Pari-2-2-9.exe|
  http://pari.math.u-bordeaux.fr/pub/pari/windows/Pari-2-2-9.exe,
   |5431 KBy, Dec 22 19:09:49 2004|
  |md5sum: 91c43064500b0d3f9e462dcef70dc6fe Pari-2-2-9.exe |
eu cheguei ao seguinte resultado empirico:
F_p(x) = p*x*(x+1)*(x^2+x+1)^n*G(x), onde G(x) eh um polinomio 
irredutivel sobre Q e n = 1 ou 2, dependendo de p. Mais precisamente:

n = 1 se p = 5, 11, 17, 23, 29 e 41
n = 2 se p = 7, 13, 19, 31, 37 e 43
Perguntas:
1) A fatoracao acima ocorre para cada primo p ou serah que G(x) eh 
redutivel para algum p?

2) Os primos para os quais n = 1 sao justamente os primos da forma 6k-1?
 

[]s, Claudio.
=
Instruções para entrar na lista, sair da lista e usar a lista em
http://www.mat.puc-rio.br/~nicolau/olimp/obm-l.html
=


[obm-l] Problemas diversos

2005-03-12 Por tôpico Marcio M Rocha
Boa tarde a todos!
Gostaria de uma ajuda com os seguintes problemas (não é necessário 
resolver, só uma idéia já é o bastante)

1) Se é que é possível, como fatorar (x + y)^7 - x^7 - y^7 sem usar 
expansão binomial?

2) Uma peça retangular é formada por quadrados, todos do mesmo tamanho. 
Existem 1274 quadrados numa direção e 990 na outra (suponho que esteja 
se referindo à horizontal e vertical). Traçando-se a diagonal desse 
retângulo, em quantas peças quadradas ela toca?

Muito obrigado desde já.
Márcio.
=
Instruções para entrar na lista, sair da lista e usar a lista em
http://www.mat.puc-rio.br/~nicolau/olimp/obm-l.html
=


Re: [obm-l] Raciocinio logico

2005-03-12 Por tôpico Marcio M Rocha
Oi, Ana.
Aqui está a sistematização do raciocínio do Bruno:
Seja x o número de manhãs com chuva, y o número de tardes com chuva e z 
o número de dias em que não choveu nem de manhã e nem à tarde. Sabemos 
que nunca houve um dia inteiro chuvoso (observação 2). Sendo assim:

x + y + z = n
x + y = 7 (observação 1)
x + z = 5 (observação 3)
z + y = 6 (observação 4)
Somando as três últimas equações você tem: 2(x + y + z) = 18, e daí, x + 
y + z = 9. Logo, são 9 dias.

Abraços.
Márcio.
Anna Luisa wrote:
Oie!
Quem sabe raciocínio lógica pra dar uma maozinha aki?
Depois de n dias de férias, um estudante observa que:
(1) Choveu 7 vezes, de manhã ou à tarde
(2) Quando chove de manhã não chove atarde
(3) Houve 5 tardes sem chuva
(4) Houve 6 manhãs sem chuva
Então n é igual a?
 
Quem souber ajuda por favor!
obrigada
Anninha.
=
Instruções para entrar na lista, sair da lista e usar a lista em
http://www.mat.puc-rio.br/~nicolau/olimp/obm-l.html
=


Re: [obm-l] Problema sobre valor minimo

2005-03-11 Por tôpico Marcio M Rocha
Valeu, Claudio. Em primeiro lugar, eu esqueci de colocar que x, y e z 
são reais positivos por hipótese.
Eu havia feito o seguinte:
xyz(x+ y + z) = 1 == xz(xy + y^2 + yz) = 1 (I)
(x + y)(y + z) = xz + xy + y^2 + yz
De (I) vem que xy + y^2 + yz = 1/xz. Sendo assim, o segundo membro de 
(II) pode ser escrito como xz + 1/xz, que, pela desigualdade das médias, 
é = 2. O que me deu dor de cabeça foi o fato dessa questão ter caído 
numa prova respeitável e o gabarito indicar 2/3 como resposta. Como você 
chegou ao mesmo que eu por outro caminho, penso que o gabarito está furado.

Obrigado de novo.
Márcio.
Claudio Buffara wrote:
Supondo que x, y e z sao reais positivos, teremos:
xyz(x + y + z) = 1 ==
y^2 + (x+z)y - 1/(xz) = 0 ==
y^2 + (x+z)y + xz - (1/(xz) + xz) = 0 ==
y^2 + (x+z)y + xz = 1/(xz) + xz ==
(x + y)(y + z) = 1/(xz) + xz = 2 quaisquer que sejam x e z positivos, com
igualdade sss xz = 1 ==
(x + y)(y + z) = 2.
O minimo de 2 eh atingido, por exemplo, com x = z = 1 e y = raiz(2) - 1.
[]s,
Claudio.
=
Instruções para entrar na lista, sair da lista e usar a lista em
http://www.mat.puc-rio.br/~nicolau/olimp/obm-l.html
=
 

=
Instruções para entrar na lista, sair da lista e usar a lista em
http://www.mat.puc-rio.br/~nicolau/olimp/obm-l.html
=


Re: [obm-l] Problema simples (trigonometria)

2004-11-29 Por tôpico Marcio M Rocha
Oi, Daniel,
Vamos dar um passo atrás a partir do ponto onde você parou.
y = (cosx + senx)(cos^2x + sen^2x - cosxsenx)
y = (cosx + senx)(1 - cosxsenx)
y = (cosx + senx)(2 - 2cosxsenx)/2
y = (cosx + senx)(3 - 1 - 2cosxsenx)/2
y = (cosx + senx)[3 - (1 + 2cosxsenx)]/2
y = (cosx + senx)[3 - (cos^2x + sen^2x + 2cosxsenx)]/2
y = (cosx + senx)[3 - (cosx + senx)^2]/2
y = a(3-a^2)/2
Daniel S. Braz wrote:
Problema 110 do livro do Iezzi (Fund. de Matematica Elementar. O
volume que trata sobre trigonometria..3 ou 4..não me lembro).
se cos x + sen x = a ; y = cos^3 x + sen^3 x. Quanto vale y ?
Eu já tentei de várias formas..mas na maioria cheguei a alguma coisa do tipo:
y = (cosx + senx)(cos^2x - cosxsenx + sen^2x)
  = a(a^2 - 3cosxsenx)
  = ??
Ai não tive mais idéias (falta de criatividade, talvez)..Mas, como
achar cosxsenx ?
A resposta dada no livro é y = a(3 - a^2)/2
[]s
daniel.
 

=
Instruções para entrar na lista, sair da lista e usar a lista em
http://www.mat.puc-rio.br/~nicolau/olimp/obm-l.html
=


[obm-l] Interpolação Bilinear

2004-11-27 Por tôpico Marcio M Rocha
Bom dia a todos!
Estou procurando material sobre interpolação bilinear. Onde posso encontrar?
Muito obrigado.
Márcio.
=
Instruções para entrar na lista, sair da lista e usar a lista em
http://www.mat.puc-rio.br/~nicolau/olimp/obm-l.html
=


[obm-l] Pesquisas acadêmicas no Google

2004-11-25 Por tôpico Marcio M Rocha
Olá, pessoa!
Repasso a vocês uma mensagem que me chegou sobre o Google. Creio que 
será útil a todos. Abraços.

O site de busca Google lançou nesta quinta-feira o Google Scholar
(http://scholar.google.com/ ), produto especializado em buscas
acadêmicas, como teses de doutorado e informes técnicos. Google
Scholar abrange um ampla gama de material, da informática a físicam em
pregando algoritmos especialmente desenhados para o mundo acadêmico.
Segundo o engenheiro responsável pelo projeto, Anurag Acharya, o
serviço, que é gratuito, a princípio não contará com publicidade,
acrescentando que o objetivo do Google é ajudar a comunidade acadêmica.
O produto é fruto da colaboração da companhia com várias publicações
acadêmicas e científicas, como a revista Nature, e se aproveita do
fato de a maioria dessas publicações terem uma edição online.
=
Instruções para entrar na lista, sair da lista e usar a lista em
http://www.mat.puc-rio.br/~nicolau/olimp/obm-l.html
=


Re: [obm-l] algebra linear

2004-11-22 Por tôpico Marcio M Rocha
a) F(x,y) = (x+y,x) e F(a,b) = (a+b, a)
Assim, *F[(x,y)+(a,b)]* = F(x+a,y+b) = (x+a+y+b, x+a) = *F(x,y) + F(a,b)*
*F[k.(x,y)]* = F(kx,ky) = (kx+ky,kx) = k.(x+y,x) = *k.F(x)*
b) F(x,y,z) = (2x-3y+4z) e F(a,b,c) = (2a-3b+4z)
Assim, *F[(x,y,z)+(a,b,c)]* = F(x+a, y+b, z+c) = (2x+2a-3y-3b+4z+4c) = 
(2x-3y+4z)+(2a-3b+4c) = *F(x,y,z) + F(a,b,c)
F[k.(x,y,z)]* = F(kx, ky, kz) = 2kx-3ky+4kz = k.(2x-3y+4z) = *k.F(x,y,z)*

andrey.bg wrote:
Mostre que as seguintes Tranformações  F são Lineares:
 

a)- F: R^2^R^2, definidas como F(x,y)=(x+y,x)
b)-F: R^3---R, definidas como F(x,y,z)=(2x-3y+4z)
=
Instruções para entrar na lista, sair da lista e usar a lista em
http://www.mat.puc-rio.br/~nicolau/olimp/obm-l.html
=


[obm-l] Álgebra linear aplicada

2004-11-19 Por tôpico Marcio M Rocha
Olá pessoal,
Alguém poderia me indicar uma boa referência em álgebra linear com 
ênfase em aplicações? Dou preferência a livros em português, mas pode 
ser em inglês também.

Obrigado.
Márcio.
 

=
Instruções para entrar na lista, sair da lista e usar a lista em
http://www.mat.puc-rio.br/~nicolau/olimp/obm-l.html
=


Re: [obm-l] Álgebra linear aplicada

2004-11-19 Por tôpico Marcio M Rocha
Fábio,
Obrigado pela indicação.
Vou comprar o do Anton. Eu vi o do Strang na Amazon por 106 dólares, mas 
aí fica meio complicado!

Fabio Niski wrote:
O livro do Anton e do Strang.
O do Anton foi traduzido para o portugues.
Marcio M Rocha wrote:
Olá pessoal,
Alguém poderia me indicar uma boa referência em álgebra linear com 
ênfase em aplicações? Dou preferência a livros em português, mas pode 
ser em inglês também.

Obrigado.
Márcio.
 

= 

Instruções para entrar na lista, sair da lista e usar a lista em
http://www.mat.puc-rio.br/~nicolau/olimp/obm-l.html
= 


=
Instruções para entrar na lista, sair da lista e usar a lista em
http://www.mat.puc-rio.br/~nicolau/olimp/obm-l.html
=
=
Instruções para entrar na lista, sair da lista e usar a lista em
http://www.mat.puc-rio.br/~nicolau/olimp/obm-l.html
=


Re: [obm-l] +++duvidas

2004-11-19 Por tôpico Marcio M Rocha
Além do expoente igual a zero, basta a base ser igual a 1, ou seja,
x^2 - 5x + 7 = 1
x^2 - 5x + 6 = 0
E aí estão os outros dois valores. (x = 2 e x = 3)
aryqueirozq wrote:
Quantos números reais satisfazem a equação
(x^2 -5x+7)^x+1 =1 ?
a) 0b) 1c) 2d) 3 e) 4
 

 

só achei -1 como resposta, eo gabarito está dizendo que eh a letra D 
como resposta.Quais as outras solucoes?

=
Instruções para entrar na lista, sair da lista e usar a lista em
http://www.mat.puc-rio.br/~nicolau/olimp/obm-l.html
=


[obm-l] Construção de um Quadrilátero Inscritível

2004-11-04 Por tôpico Marcio M Rocha




Pessoal,

Depois de hesitar muito, tentei resolver o problema 
proposto pelo prof. Wagner e que foi ressuscitado pelo Cláudio.Gostaria 
que vocês comentassem essa"tentativa" de solução. Mesmo que esteja tudo OK 
eu achei muito trabalhosa, mas não consegui ver outra coisa. Aqui 
vai:

Supondo o quadrilatero construído, a respeito da 
diagonal AC podemos escrever:

(AC)^2 = (AB)^2 + (BC)^2 - 2.AB.BC.cosB, 
e
(AC)^2 = (AD)^2 + (DC)^2 - 
2.AD.DC.cosD

Como B e D são suplementares temos que cosB = -cosD 
= k.

Comparando as duas expressões e isolando k, 
temos:

k = [(AB)^2 + (BC)^2 - (AD)^2- (DC)^2] / 
[2.(AB.BC + AD.DC)]

Suponhamos que B seja um ângulo agudo (para que k 
seja positivo)

Adotando como unidade o raio de uma circunferência 
qualquer, o numerador da expressão acima e construtível, o mesmo acontecendo com 
o denominador. Assim sendo, podemos construir k.

Após construído, transferimos o segmento 
kpara acircunferência cujo raio adotamos como unidade, e encontramos 
o valor do ângulo B. Agora o problema se resume a construir um triângulo ABC 
conhecendo seus lados AB e BC e o ângulo B, o que pode ser feito sem 
dificuldade.

Como as mediatrizes dos lados AB e BC 
intersectam-se no centro da circunferência circuscrita ao quadrilátero, ela pode 
ser construída e, a partir daí, é só marcar os segmentos CD e DA.

Saudações.

Márcio.


Re: [obm-l] 0,9999...=1?

2004-10-19 Por tôpico Marcio M Rocha



Olá, Gabriel.

Gostaria de dar uma outra explicação, além da que 
foi dada pelo Bernardo. Ela foi dada pelo Prof. Paulo Cezar Carvalho no Curso de 
Aperfeiçoamento de Professores, realizado este ano no IMPA.

Primeiro, temos que 0,999...  = 1 ( 
=: "menor do que ou igual a")

Suponha que 0,999...  1

Observe que:

1 - 0,9 = 0,1
1 - 0,99 = 0,01
1 - 0,999 = 0,001
.
Isso significa quea distância entre 0,999... 
e 1 pode tornar-se tão pequena quanto você queira, bastando, para isso, tomar 
uma quantidade de casas decimais conveniente. Essa distância, então, é sempre 
menor que qualquer número real. Sendo assim, 0,999... não pode ser estritamente 
menor que 1. Logo, 0,999... = 1.

Qualquer dúvida, é sóescrever.

Márcio.

- Original Message - 

  From: 
  gabriel 
  
  To: obm-l 
  Sent: Tuesday, October 19, 2004 7:19 
  PM
  Subject: [obm-l] 0,...=1?
  
  Olá
  
  há algum tempo eu li alguns e-mails aki na lista q tratavam do seguinte 
  tema:
  
  0,99...=1?
  
  Será q alguem poderia me explicar mais detalhadamente o 
assunto?
  
  
  Gabriel.


[obm-l] Soma de números primos

2004-10-13 Por tôpico Marcio M Rocha




 Boatarde a 
todos.

 Gostaria da ajuda 
de vocês com o seguinte problema:

 
“Demonstre que a soma de todos os números primos entre 1 e 2004 é menor que 
667222.”

 Tentei um caminho 
destrutivo, eliminado alguns números que não são primos:

a) 
Da 
seqüência 1, 2, 3, ..., 2004, retirei o 1 e os números pares maiores que 
2.
b) 
Calculei 
a soma S dos termos da seqüência 
restante

S = 2 + 3 + 5 + 7 + 9 +...+ 
2003

obtendo S = 1 004 005.

c) 
Da 
seqüência anterior, eliminei os múltiplos de 3 maiores que 3. Como a soma desses 
múltiplos é 334 665, 
a soma

S1 = 2 + 3 + 5 + 7 + 11 + 13 + 17 + 
... + 2003

vale

S1 = S – 334 665 = 669 
340.

Como a seqüência 2, 3, 5, 7, 11, 
..., 2003 é formada ainda por números compostos, basta que eu retire alguns 
deles, lembrando apenas de não retirar nenhum múltiplo de 3. Retiro, então, 1963 
= 13 x 151 e 155 =5 x 31, e a soma dos números restantes fica igual a 667 
222. Como ainda há números compostos, está claro que a soma dos primos entre 1 e 
2004 deve ser menor que 667 222.

Está tudo OK? 
Alguémpoderiadarum caminho melhor?

Abraços,

Márcio Rocha.

P.S. Embora reconheça que muitos 
participantes da lista não necessitem, gostaria de pedir em meu nome ( e talvez 
no de outros), que as soluções, sempre que possível, viessem acompanhadas das 
"motivações", para que aqueles que lêem não fiquem com a sensação de "coelho 
tirado da cartola". Peço isso porque li um artigo de Miguel de 
Guzmán onde ele diz queEuler, em sua obra,"colocava-se inicialmente na ignorância do 
temae dos métodos queiriaempregar, para começar en condicões 
de igualdade con aquele a quem trata de conduzir pelo caminho,ajudando-o a 
ver as dificuldades que elemesmo encontrou, levando-o,às vezes, 
porcaminhos equivocados queele mesmo havia percorrido antes, a fim 
de que aprenda também dos equívocos". (O artigo completo em espanhol está em www.campus-oei.org/oim/saladelectura.htm, 
sob o título "O papel do matemático en la educación 
matemática"

Se não estiver fora do tema, 
poder-se-ia discutir também estratégias de solução, como as apresentadas no 
Problem Solving Strategies.

Desculpem se escrevi 
demais.


[obm-l] SEQÜÊNCIA DOS DIVISORES POSITIVOS

2004-10-05 Por tôpico Marcio M Rocha



Olá, pessoal.

Já que estão falando sobre divisores, vejamos o 
seguinte:
Seja N um número natural com uma quantidade ímpar 
de divisores positivos. Então N é quadrado perfeito, OK? Agora duas 
perguntas:

1) Ordenando crescentemente esses divisores de N, o 
termo que ocupa o lugar central é sempre igual a raiz quadrada de 
N?

2) Quando N possui apenas um fator primo, seus 
divisores formam uma PG cuja razão é o próprio fator primo. Pórém, quando isso 
não acontece,a razão entre um termo e seu antecessor não é contante. 
Apesar disso, há uma repetição dessas razões. Observem os exemplos 
abaixo:


 a) Div(36) = 1, 2, 3, 4, 6, 9, 12, 18, 36

 36/18 = 2
 18/12 = 1.5
 12/9 = 1.333...
 9/6 = 1.5
 6/4 = 1.5
 4/3 = 1.333...
 3/2 = 1.5
 2/1 = 2

 b) Div(100) = 1, 2, 4, 5, 10, 20, 25, 50, 
100

 100/50 = 2
 50/25 = 2
 25/20 = 1.25
 20/10 = 2
 10/5 = 2
 5/4 = 1.25
 4/2 = 2
 2/1 = 2

 c) Div(225) = 1, 3, 5, 9, 15, 25, 45, 75, 
225

 225/75 = 3
 75/45 = 1.666...
 45/25 = 1.8
 25/15 = 1.666...
 15/9 = 1.666...
 9/5 = 1.8
 5/3 = 1.666...
 3/1 = 3

Não quero 
encher a lista com cultura inútil, por isso, se for RELEVANTE,é possível 
generalizar algo sobre isso?

Abraços.

Márcio.


[obm-l] Re: [obm-l] Dúvida

2004-10-03 Por tôpico Marcio M Rocha
Douglas, a sua pergunta foi ótima, porque dizer simplesmente que 0! =1
por definição pode dar a idéia de que é algo imposto, e que poderiam ter
definido 0! como sendo igual a qualquer coisa, quando não é esse o caso.
Penso que a idéia básica é a mesma de quando se define que qualquer número
não nulo elevado a zero é 1. Ou seja, não cair em certas dificuldades. Por
exemplo, o binomial (n, n) é, sem dúvida, igual a 1, já que exprime o número
de combinações de n objetos tomados n a n. Calculando-o via fatorial,
obtemos 0! no denominador. O único valor plausível para 0! é 1.
 Creio que repostas melhores aparecerão, mas, no pouco tempo que tive
para escrever, foi o melhor que pude conseguir.
Abraços.
Márcio.
- Original Message - 
From: Douglas Drumond [EMAIL PROTECTED]
To: [EMAIL PROTECTED]
Sent: Sunday, October 03, 2004 7:09 PM
Subject: Re: [obm-l] Dúvida


   Gostaria de saber por que 0! = 1.
  Por definicao.

 Mas qual foi a motivação para definir 0! = 1 ?

 []'s

 Douglas

 =
 Instruções para entrar na lista, sair da lista e usar a lista em
 http://www.mat.puc-rio.br/~nicolau/olimp/obm-l.html
 =

=
Instruções para entrar na lista, sair da lista e usar a lista em
http://www.mat.puc-rio.br/~nicolau/olimp/obm-l.html
=